You are on page 1of 5

Indian Institute of Technology Kanpur

Department of Mathematics and Statistics


Single Variable Calculus (MTH 111)
Hints for Exercise Sheet 6
1. To be discussed in the tutorial session
P∞
In what follows, the phrase
P∞ ‘Discuss the convergence of the series n=1 an ’ means you have to
determine whether or not n=1 an converges, and if it does, then whether the convergence is absolute.

(1) Let {an }∞ ∞


n=1 be a real sequence and N ∈ N. Define a sequence {bn }n=1 by bn = aN +n−1 , for
X∞ ∞
X
∞ ∞
all n ∈ N. We call {bn }n=1 and bn the N -th tail of {an }n=1 and an respectively. It is
n=1 n=1

X ∞
X
customary to denote {bn }∞ ∞
n=1 by {an }n=N and bn by an . Show the following:
n=1 n=N
(a) {an }∞ ∞
n=1 converges if and only if {an }n=N converges, and in that case the limit will be
same.
∞ ∞ ∞
X X X
(b) an converges if and only if an converges, and in that case, one has an =
n=1 n=N n=1

X
a1 + · · · + aN −1 + an .
n=N
Note: In view of (1b), the hypothesis of the ‘Comparison test’ can be relaxed a little as follows:

Let {cn }∞ ∞
n=1 and {dn }n=1 be two series of nonnegative
P∞ terms. suppose
P∞that there exits
N ∈P N such that, ∀n ≥ N ,P cn ≤ dn . Then n=1 dn converges =⇒ n=1 cn converges,
and ∞ c
n=1 n diverges =⇒ ∞
n=1 dn diverges.

Solution. Both (1a) and (1b) follow directly from the definition of convergence of a sequence.
Hence details are left to you.
def
(2) Let f : [0, 1] −→ R. Consider an = f n1 − f n+1 1
 
, for all n ∈ N. Show the following:
X∞
(a) If f is continuous at 0 then an converges.
n=1

X∞
Solution. Since the series an is telescopic, it converges if and only if so does the
 ∞ n=1
sequence f n1 n=1 , which is evident from the continuity of f at 0.



X

(b) If f is differentiable on (0, 1) and and the function f is bounded then an converges
n=1
absolutely.
Solution. Let M ≥ 0 be such that, ∀x ∈ (0, 1), |f ′ (x)| ≤ M . Let n ∈ N. From MVT,
we obtain ξn ∈ n+1 1
, n1 such that, |an | = f n1 − f n+11 = |f ′ (ξn )| 1 ≤ M 1 .
n(n+1) n(n+1)

X 1
Since converges, from Comparison test, one obtains the absolute convergence
n=1
n(n + 1)

X
of an .
n=1

1
2


X
(3) Let an be a series such that for any n ∈ N, an + 1 ̸= 0. Show the following:
n=1
∞ ∞
X X an
(a) an is absolutely convergent =⇒ so is .
n=1 n=1
an + 1

X
Solution. From the absolute convergence of an , one obtains N ∈ N such that, for all
n=1
n ≥ N , |an | ≤ 21 . This implies that, for all n ≥ N , |an + 1| ≥ 1 − |an | ≥ 21 . It now
follows that, for all n ≥ N , |a|an +1|
n|
≤ 2|an |. The comparison test now yields the absolute

X an
convegrence of .
n=1
a n + 1
∞ ∞
X X an
(b) If an ≥ 0, for all n ∈ N, and an diverges then so does .
n=1 n=1
an + 1
Solution. We consider the following two cases:

Case 1: There exists N ∈ N such that, for all n ≥ N , an ≤ 1. It follows that, for all
an an
n ≥ N, ≥ . Now the divergence is clear in view of the Comparison test.
1 + an 2
Case 2: The opposite of Case 1, i.e., for all N ∈ N, there exists n ≥ N such that an ≥ 1.
an 1
Observe that, if an ≥ 1, then ≥ . From this we obtain that, for all N ∈ N, there
1 + an 2  ∞
an 1 an
exists n ≥ N such that ≥ . Therefore does not converge to 0, and
1 + an 2 1 + an n=1

X an
thus cannot converge.
a +1
n=1 n

(4) Let p, q ≥ 1 be any real numbers with p < q. Suppose that {an }∞ n=1 be a sequence of

X ∞
X
nonnegative numbers with the property that apn converges. Show that aqn is convergent.
n=1 n=1

X
Solution. Since apn converges, there exists N ∈ N such that, for all n ≥ N , we have
n=1
apn ≤ 1. This implies that, for all n ≥ N , an ≤ 1. It now follows that
aqn ≤ apn , ∀n ≥ N. (1.1)
X∞
In view of the Comparison test, the convergence of aqn is now immediate from (1.1).
n=1

(5) (a) Let {an }∞ ∞


n=1 and {bn }n=1 be two sequence of positive reals such that, for all n ∈ N,
∞ ∞
an+1 bn+1 X X
≤ . Show that, if bn converges then same holds for an .
an bn n=1 n=1
 ∞
an
Sketch of the proof. Does this not show that is decreasing? Can you now
bn n=1
see some familiar test to apply to this case?

X nn−2
(b) Test the series for convergence.
n=1
en n!
3

def 1
Sketch of the proof. Can you see whether (5a) can be used with bn = n2
?

X 1
(6) Find all p > 0 for which the series converges.
n=3
n log n(log(log n))p
Sketch of the proof. Use Cauchy’s condensation test, Recall that, we have discussed in the

X 1
lecture, for which values of p > 0, converges. Proceed likewise.
n=3
n(log n)p

∞ ∞ √
X
1
X an
(7)∗ Let an be a convergent series with nonnegative terms. Show that, for every p > 2
,
n=1 n=1
np

X ∞
X
1
converges. Does the same hold for all p ∈ (0, 2
]? (Hint: If two series xn and yn of
n=1 n=1

X √
nonnegative terms converge, can you see whether or not xn yn converges?)
n=1
∞ ∞ ∞ √
1
X X 1 X an
Solution. Let p > 2
.
Since an and 2p
are convergent, so is p
. Assume now
n=1 n=1
n n=1
n
∞ ∞
X 1 X 1
0 < p ≤ 21 . Clearly the series 2
converges, but 1 does not converge
n=2
n(log n) n=2 n 2
+p
(log n)
in view of Cauchy’s condensation test.
a1 + · · · + an
(8)∗ Let {an }∞
n=1 be a sequence with an − −−→ ℓ, where ℓ ∈ R. Define bn = , ∀n ∈ N.
n→∞ n
(a) Show that bn −−−→ ℓ. (Hint: It suffices to prove the statement when ℓ = 0. Why? So
n→∞
assume now ℓ = 0. Let M > 0 be such that, for all n ∈ N, |an | ≤ M . Observe that, for
any m, n ∈ N with n > m, one has

a1 + · · · + an |a1 | + · · · + |am | |am+1 | + · · · + |an |
≤ +
n n n
mM |am+1 | + · · · + |an |
≤ + . (1.2)
n n
Now, given any ε > 0, taking n large enough, we can make the terms |am+1 |, . . . , |an | < ε.)

Solution. Let ε > 0. Then there exists m ∈ N such that, for all n ≥ m, |an | < 2ε . Now
from (1.2), we see that, for all n > m,

a1 + · · · + an mM |am+1 | + · · · + |an | mM n−m mM ε
≤ + < + ε< + . (1.3)
n n n n n n 2
mM ε
From Archimedian property, there exists n0 ∈ N such that < . Clearly, this gives
n0 2
us
mM ε
∀n ≥ n0 , < . (1.4)
n 2
def
Consider
N = max{m, n0 }. It follows from (1.3) and (1.4) that, for all n ≥ N ,
a1 + · · · + an ε ε
< + = ε.
n 2 2

X
(b) Assume that {an }∞
n=1 is decreasing. Discuss the convergence of the series (−1)n+1 bn .
n=1
4


X
Sketch of the proof. If ℓ ̸= 0, certainly (−1)n+1 bn cannot converge. Now we assume
n=1
ℓ = 0. From {an }∞ ∞
n=1 is decreasing, show that {bn }n=1 is decreasing. Now from Leibniz’s
X∞
test, the convergence of (−1)n+1 bn is immediate.
n=1


X 1
(9) Discuss the convergence of the series (−1)n+1 sin .
n=1
n
π
Solution. Since 0 < 1 < ,
the sine function is decreasing on [0, 1], and also we have
2

X 1
1
sin n −−−→ 0. Hence by Leibniz’s test, (−1)n+1 sin converges. This convergence is not
n→∞
n=1
n
1 ∞
sin X1
absolute because 1 n −−−→ 1 and diverges.
n
n→∞
n=1
n


X
(10) Find the radius of convergence of the power series an xn , where
n=1
(
1
def n
if n is a perfect square
an =
0 otherwise.

Solution. Let n ∈ N. Then there exists k ∈ N such that k 2 ≤ n < (k + 1)2 . Hence, for any
ρ ≥ 0, we have
k 2 n k+1 r2
X ρr X
i
X ρ
≤ ai ρ < . (1.5)
r=1
r2 i=1 r=1
r2

X
From (1.5), it is clear that, for any |x| ≤ 1, |ai ||x|i is convergent.
i=1
∞ 2
X ρr
Let ρ > 1. From Ratio test, it is easy to see that diverges, From this it follows
r=1
r2

X
that, if |x| > 1, then the series an xn cannot converge, since otherwise we would have got
n=1

X
|ai ||y|i is convergent, for any y ∈ (−|x|, |x|).
i=1

2. Additional exercises

an+1
(1)∗ (a) Suppose that {an }∞
n=1 is a sequence of nonzero real numbers such that an −−−→ ℓ,
n→∞
1
where ℓ ∈ R. Show that |an | n −−−→ ℓ.
n→∞
Does (1a) not show that, if the ratio test yields a given series is absolutely convergent or
divergent then so does root test? This justifies the claim that root test is stronger than the
ratio test.
n
(b) Compute lim 1 .
n→∞ (n!) n
5


X
Let an (x − a)n be a power series with radius of convergence R ∈ (0, ∞]. Define
n=0

X
f (x) = an (x − a)n , ∀x ∈ (a − R, a + R). (2.1)
n=0

The aim of the following exercises (2) and (3) is to establish that the function f defined above in
(2.1) is differentiable, and one further has

X

∀x ∈ (a − R, a + R), f (x) = nan (x − a)n−1 . (2.2)
n=1

(2) Show that, for all α, β ∈ R and n ∈ N,


|(α + β)n − αn | ≤ n|β|(|α| + |β|)n−1 . (2.3)

(3)∗ Let x ∈ (a − R, a + R) and ε > 0. Fix r > 0 such that |x − a| < r < R. Observe that, for any
N ∈ N and 0 < |h| < r − |x − a|, the following holds:

∞ ∞
X (x + h − a)n − (x − a)n X
an ≤ n|an |(|h| + |x − a|)n−1

h


n=N +1 n=N +1
X∞
≤ n|an |rn−1 .
n=N +1


X ε
(a) Show that, there exists N ∈ N such that n|an ||x − a|n−1 < , and
n=N +1
4


X (x + h − a)n − (x − a)n ε
an < , whenever 0 < |h| < r − |x − a|.

h 4


n=N +1

(b) Let N be as in (3a). Show that, there exists δ ∈ (0, R − |x − a|), such that, for all
0 < |h| < δ, one has

N
(x + h − a)n − (x − a)n

ε
X
an − n(x − a)n−1 < .

h 2


n=1

(c) Conclude (2.2) from (3a) and (3b).

You might also like